LSAT and Law School Admissions Forum

Get expert LSAT preparation and law school admissions advice from PowerScore Test Preparation.

User avatar
 Dave Killoran
PowerScore Staff
  • PowerScore Staff
  • Posts: 5850
  • Joined: Mar 25, 2011
|
#41151
Complete Question Explanation
(The complete setup for this game can be found here: lsat/viewtopic.php?t=15627)

The correct answer choice is (D)

Quickly glancing at the initial diagram shows that G can be ranked as low as seventh (with only J and H behind G), and as high as fifth (with L, N, J, and H behind G, now necessarily in that order). Given the latitude of L and N, it is reasonable to expect that G could sixth as well, and that the correct answer is fifth, sixth, or seventh (which is answer choice (D)).

However, for further, incontrovertible proof, consider the following three hypothetical sequences:

Feb92_Game_#1_#6_diagram 1.png
The first sequence proves that G could possibly be ranked fifth, the second sequence proves that G could possibly be ranked seventh, and the third sequence proves that G could possibly be ranked sixth. Since “fifth,” “sixth,” and “seventh” must all be in the complete and accurate list of G’s possible ranks, answer choices (A), (B), and (C) can all be eliminated. G cannot possibly be ranked eighth since G’s salary must be higher than both J’s salary and H’s salary. Hence, answer choice (E) can be eliminated. Answer choice (D) is thus proven correct by process of elimination.
 andriana.caban
  • Posts: 142
  • Joined: Jun 23, 2017
|
#36363
Hi! I need help with understanding why number 6 is D. I understand why Glassen can be placed in position five and seven, however, not for 6.

If G was placed in 6, where would L-N go? : K-I-F-M (so the first four places are out), J-H (so the last two places are out). If G were in six, J-H would follow with one space for 9, but L-N needs to be together?
 Luke Haqq
PowerScore Staff
  • PowerScore Staff
  • Posts: 722
  • Joined: Apr 26, 2012
|
#36456
Hi andriana!

You're definitely right to work through this one by eliminating the first 4 spaces (because K, I, F, and M must come before G) and eliminating the last 2 spaces (because J and H must come after G).

To your question,
If G was placed in 6, where would L-N go?
the short answer is, if the other spots were taken up, then L would go in 5 and N would go in 7.

Your diagram should look like the following:

I :longline: F :longline: M :longline: G :longline: J :longline: H
K <
L :longline: N

I know that doesn't show up perfectly, but K should be more clearly to the left of I and L. Within that diagram, we don't actually know the relationship between G and L/N. We know that L and N have to be after K, but we don't know their interactions with any other variables. Further, we know that N comes after L, but they don't have to be right next to each other. So, for example, the following order could work:

K, I, F, M, G, J, L, H, N

That should fit all the rules, and as you can see, L and N don't need to be together. J and H also do not need to be last, nor do they need to occur right next to each other.

Rather, we know that a complete and accurate list of places G could be in (if no two salaries are the same) would be spots 5, 6, or 7 for the reasons you noted (because a minimum of K, I, F, M comes before G and a minimum of J, H comes after).
 concrottrox11@gmail.com
  • Posts: 29
  • Joined: Dec 07, 2021
|
#92484
Hi,

Why couldn't N hypothetically be anywhere after L when thinking of hypothetical sequences for this question? Why does it have to be either last or 3rd?
 Robert Carroll
PowerScore Staff
  • PowerScore Staff
  • Posts: 1787
  • Joined: Dec 06, 2013
|
#92505
concrottrox,

Dave's diagrams at the top are hypotheticals - he's not trying to show everywhere N can go, because that's not relevant to this question. He's trying to prove that G can go 3 different places. The placement of every other variable in this game is relevant, for this question, only to answer the question of where G can go. So certainly N could go many other places. However, really the only relevant placements of any variable for THIS question would be "after G" or "before G":

Must be before G: K, I, F, M

Must be after G: J, H

Can go either way: L, N

Either L and N are both after G, or G is somewhere between then, or they are both before G. That results in three placements for G, specifically what answer choice (D) says (5th, 6th, 7th). Where you put L and N specifically is not really important - their relations to G are the only important things here.

Robert Carroll

Get the most out of your LSAT Prep Plus subscription.

Analyze and track your performance with our Testing and Analytics Package.